Download as pdf or txt
Download as pdf or txt
You are on page 1of 25

AIMTED

2017 Practice Exam Questions and Answers


A 70-year-old male presents to the ED with chest pain that
started 40 minutes ago. The chest pain came on at rest, is
dull in nature and located centrally. Nothing makes the pain
better or worse. He is also short of breath and nauseous. The
patient has a past medical history of stable angina and
hypertension, which is managed with GTN and ramipril. The
Percutaneous intervention (PCI)
patient has a 40 pack-year smoking history and consumes 2
standard drinks of alcohol per day. His vital signs are HR
Explanation: This 70-year-old patient has a classical history and
98, BP 144/94, RR 20, O2 sats 96% RA and 37.2˚C.
ECG findings suggesting an inferior ST-elevation myocardial
Physical examination reveals no abnormalities aside from
infarction (STEMI). It has been less than 1 hour since symptom
the patient being in obvious pain, clutching the centre of his
onset. PCI is available within 1 hour. Accordingly, PCI is the

17
chest. Otherwise physical examination is non-contributory.
most appropriate definitive treatment option. For further
ECG reveals sinus tachycardia and 3 mm of elevation of
information, see the National Heart Foundation of Australia
leads II, III and aVF. Bloods are sent off for initial
guidelines.
investigations. You are in a large tertiary hospital and
percutaneous intervention is available within 1 hour.
Thrombolysis is available within 15 minutes.

20
What is the most appropriate definitive management for this
patient?
A 71-year-old male presents to the ED with shortness of
breath for the last few hours. He has also been feeling tired Echocardiogram
recently. The patient has a past medical history of advanced
non-Hodgkins lymphoma. Vital signs are HR 130, BP Explanation: The combination of Beck’s triad along with a
D
106/66, RR 20, O2 sats 97% RA and 37.2˚C. Physical history suggesting the possibility of pericardial metastases makes
examination reveals distended neck veins and muffled heart cardiac tamponade the most likely diagnosis. The diagnostic
sounds. investigation of choice for cardiac tamponade is an
TE
echocardiogram
What is the most appropriate diagnostic investigation?
A 25-year-old female presents to the GP with a rash on her
neck that started over the past month. The rash is mildly
itchy. The patient has no significant past medical history. Allergic contact dermatitis
Physical examination reveals an erythematous papular rash
M

with small vesicles that has quite well defined margins. The Explanation: The distribution of this rash characteristic of an
rash is in the distribution of the metal necklace that she is allergic contact dermatitis. In this instance, the type 4
wearing. Otherwise physical examination is non- hypersensitivity reaction is likely due to nickel in the necklace.
contributory. Other classic examples include watches and belt buckles.
AI

What is the most likely diagnosis?


A 48-year-old male presents to the GP with a lesion on his
skin. He has a violaceous papule on the left side of his
HHV8
forehead. Further inspection reveals a similar coloured
macule in his hairline. He reports that neither lesion is itchy
©

Explanation: The lesions present in this case are consistent with


or painful. The patient has a past medical history of HIV,
Kaposi sarcoma. These lesions are caused by human herpes virus
which is managed with highly active antiretroviral therapy.
8 (HHV8) and normally described in immunosuppressed
However, the patient is known to be poorly compliant. Vital
individuals. Although classically described in HIV patients, they
signs are stable and 37.0˚C. Physical examination reveals no
may occur in other immunocompromised patients, such as those
other abnormalities.
who have received organ transplants.
Which virus is responsible for this lesion?

1 © AIMTED 2017. All Rights Reserved. See website for disclaimer.


http://www.aimted.com
A 38-year-old male presents to the GP with bilateral pain in Serum IGF-1
his knees and feet. The pain is constant throughout the day.
He thinks the pain in his feet may be because of his new Explanation: The presence of increasing foot size and hand size
shoes he bought recently because his old ones didn’t fit with joint pain and widely spaced teeth is suspicious for
anymore. He also reports that his wedding ring does not fit acromegaly. Initial investigation for suspected acromegaly may
anymore either. The patient has no significant past medical include serum insulin-like growth factor 1 (IGF-1) testing as this
history. Vital signs are stable and 37.0˚C. Physical is a sensitive test for acromegaly (and a normal result is effective
examination reveals widely spaced teeth. for ruling out the diagnosis). Elevated growth hormone following
an oral glucose tolerance test is a more specific test that may
What is the most appropriate investigation? follow IGF-1 testing.

A 35-year-old female presents to the ED with abdominal


pain that started 4 hours ago. The abdominal pain is poorly
localised. She also complains of nausea and vomiting, IV hydrocortisone

17
weakness, muscle cramps over the past few days. The
patient has no significant past medical history. The patient is Explanation: The combination of the abdominal pain,
tachycardic, hypotensive and afebrile. Physical examination nausea/vomiting, weakness, muscle cramps and electrolyte
reveals decreased skin turgor and sunken eyes. Otherwise disturbances make adrenal insufficiency the most likely diagnosis.
physical examination is non-contributory. IV fluids are Hydrocortisone will temporarily correct this adrenal insufficiency
commenced. Initial investigations reveal hypoglycaemia, and improve glycaemia and potassium balance. Alternatively,

20
hyponatraemia and hyperkalaemia. dexamethasone may also be used.

What is the most appropriate next step in management?


A 42-year-old male presents to the GP with joint pain that
started 2 days ago. The joint pain is symmetrical and
Ross river fever
involving his hands, knees and ankles. The pain is severe
D
and limiting his daily activities. He has also been feeling
Explanation: The combination of severe symmetrical joint pain
tired. The patient has no significant past medical history. He
with fever and rash following exposure to mosquitos supports the
recently returned from a camping holiday in northern
diagnosis of Ross river fever. Serology is required to confirm the
Queensland. Vital signs are stable and 38.2˚C. Physical
TE
diagnosis. Dengue fever classically presents with more prominent
examination reveals diffuse swelling of the affected joints
muscle aches than joint pains. However, the two conditions may
and a maculopapular rash over his trunk. Several papules are
be indistinguishable clinically. Another differential to consider
visible on his arms which he reports are mosquito bites.
would be Barmah forest virus, as this may present very similarly
There is also inguinal and axillary lymphadenopathy.
to Ross river fever.
What is the most likely diagnosis?
M

A 71-year-old male presents to the ED with rectal bleeding.


This bleeding started suddenly one hour ago and is bright Diverticulitis
red. He also complains of left lower quadrant pain. He has
never had this before and had no change in his bowel habits Explanation: This presentation with acute PR bleeding with LLQ
AI

before this. He has a past medical history of hypertension, pain and fever supports the diagnosis of diverticulitis.
which is managed with atorvastatin. There is a family Angiodysplaisa typically causes painless PR bleeding. While
history of cardiovascular disease. Vital signs are stable and hereditary haemorrhagic telangiectasia may also cause PR
37.7˚C. Physical examination reveals bright red blood on bleeding, there is little else to support this diagnosis (such as a
rectal examination and no other abnormalities. familiy history, autosomal dominant inheritance, or
telangiectasias).
©

What is the most likely diagnosis?

2 © AIMTED 2017. All Rights Reserved. See website for disclaimer.


http://www.aimted.com

A 52-year-old female presents to the GP with abdominal


pain and lethargy for the past week. The pain is located in
the right upper quadrant and dull in nature. The patient has a
Corticosteroids
past medical history of Hashimoto’s thyroiditis and type 1
diabetes mellitus, which is managed with thyroxin and
Explanation: This presentation indicates acute hepatitis with
insulin respectively. Vital signs are stable and 37.8˚C.
failure of the synthetic function of the liver (elevated bilirubin).
Physical examination reveals jaundice of the sclera.
The presence of anti-smooth muscle antibodies and a raised IgG
Subsequent investigation reveals highly elevated ALT and
supports the diagnosis of type 1 autoimmune hepatitis.
AST and elevated bilirubin. She is found to be positive for
Corticosteroids are the most appropriate treatment and may
ANA and anti-smooth muscle antibodies and has a raised
prevent fulminant liver failure.
IgG.

What is the most appropriate treatment?


A 32-year-old female presents to the GP with diarrhoea that

17
started 2-3 months ago. The diarrhoea occurs 3-4 times per
day. The stools are grey, frothy and malodorous. She also Anti-gliadin antibodies
reports that over the same time she has had abdominal
bloating, abdominal pain and lost 5kg unintentionally. The Explanation: The presence of chronic diarrhoea with steatorrhoea
patient has no significant past medical history. The patient is and weight loss accompanied by positive anti-tissue
haemodynamically stable and afebrile. Physical examination transglutaminase antibodies makes coeliac disease the most likely

20
reveals no abnormalities. Investigations reveal positive anti- diagnosis. In addition to anti-tissue transglutaminase (anti-TTG),
tissue transglutaminase antibodies. autoantibodies that may be positive in coeliac disease include
anti-gliadin and anti-endomysial antibodies.
Which of the following other autoantibodies will most likely
also be positive?
A 44-year-old male presents to the GP with recurrent chest Lifestyle modification
D
pain. The pain is burning in nature and generally occurs
following large meals. The patient has no significant past Explanation: The nature and exacerbating/alleviating factors for
medical history. He has been using antacids, which provide this patient’s chest pain support the diagnosis of gastroesophageal
some relief. Vital signs are stable and 37.0˚C. Physical reflux disease (GORD). The first step in managing GORD is
TE

examination reveals central obesity. lifestyle modification with the avoidance of trigger foods (such as
chocolate), eating smaller meals, weight loss and avoiding eating
What is the most appropriate initial treatment? and drinking in the 2 hours before going to bed.
A 23-year-old female presents to the ED with RLQ
abdominal pain that started 8 hours ago. The pain become
M

significantly more severe 30 minutes ago. Moving worsens


the pain. She reports that she has had spotting for the last
week. Her last period was 6 weeks ago (she normally has a Ectopic pregnancy
regular 28 day cycle). She is sexually active with her
AI

boyfriend of 12 months and uses condoms intermittently for Explanation: The diagnosis is supported by lower abdominal pain
contraception. She has never had a sexually transmitted in a woman of childbearing age who has had her last period 6
infection and denies symptoms of dysuria and vaginal weeks ago and spotting (who normally has a regular cycle). The
discharge. The patient has no significant past medical sexual history is also consistent with an ectopic pregnancy. Pelvic
history. The patient has a 4 pack-year smoking history and inflammatory disease is less likely as, in addition to not
consumes 3 standard drinks of alcohol per day. Vital signs accounting for her menstrual abnormalities, is typically associated
©

are HR 110, BP 104/68, RR 20, O2 sats 98% RA and with a vaginal discharge.
37.2˚C. Physical examination reveals tenderness over the
right lower quadrant and rigidity of the abdomen. Otherwise
physical examination is non-contributory. What is the most
likely diagnosis?
A 18-month-old male is brought to the GP because his X-linked recessive
parents are concerned that he bruises unusually easily and
when he gets a scratch it is slow to stop bleeding. Two of his Explanation: Haemophilia A is an X-linked recessive condition
maternal uncles have bleeding disorders. Subsequent resulting in factor VIII deficiency. Due to the X-linked nature of
investigation reveals that he has a prolonged APTT and a the disease males are typically affected and inherit the disease
deficiency of factor VIII. from a carrier mother. A son cannot inherit the disease from his
father. Females can also develop the disease, although rarely. For
What is the mode of inheritance of the most likely example, if they have an affected father and carrier or affected
diagnosis? mother (thereby becoming homozygous for the mutated gene).

3 © AIMTED 2017. All Rights Reserved. See website for disclaimer.


http://www.aimted.com

A 29-year-old female presents to the ED with a cough that is


productive for chunks of orange sputum. The cough has ABPA
been present for several days. She has also had a headache.
She has had several episodes like this over the past few Explanation: The finger-in-glove sign on a chest x-ray indicates a
months. The patient has a past medical history of cystic bronchocele (bronchiole filled with mucus), which may occur in
fibrosis and chronic sinusitis. The patient has never smoked allergic bronchopulmonary aspergillosis (ABPA). The past
and doesn’t drink alcohol. Vital signs are stable and 38.3˚C. history of cystic fibrosis and current fever, headache and orange
Physical examination reveals a wheeze bilaterally. A chest sputum support this diagnosis. The chronic sinusitis provides
x-ray reveals bilateral diffuse upper lobe interstitial further support for the presence of aspergillus. The elevated
opacities. There is increased perihilar opacity that has eosinophils and IgE support the presence of a hypersensitivity
projections giving the appearance of "a finger in a glove". reaction. ABPA is a hypersenitivity reaction to aspergillus in the
Blood tests reveal elevated eosinophils and elevated IgE. airways (as opposed to an invasive infection in invasive
Blood serology and sputum culture are pending. aspergillosis), and most often occurs in patients with cystic
fibrosis and asthma.

17
What is the most likely diagnosis?

A 32-year-old male presents to the GP with headaches and


fatigue for 48 hours. He wonders if a scratch he sustained Bartonella henselae
from his cat may be getting infected. The cat scratched him
2 weeks ago. Vital signs are stable and 37.7˚C. Physical Explanation: The combination of a cat scratch 1-3 weeks ago with

20
examination reveals 4 cm scratch on the right arm with a a pustule at the site and tender lymphadenopathy at the lymph
pustular lesion at the site of the scratch. Tender right axillary nodes draining from the site make cat scratch disease the most
lymphadenopathy is also identified. likely diagnosis. Cat scratch disease is most often caused by
Bartonella henselae.
What is the most likely causative organism?

A 37-year-old female presents to the GP with increased


D
urgency when needing to pass urine over the past 24 hours.
She has also noticed pain passing urine. The patient has no
Gram negative, bacilli
significant past medical history. She is sexually active with
her husband and uses a levonorgestrel intrauterine system
TE
Explanation: In this case the patient has presented with typical
for contraception. She is haemodynamically stable and
symptoms of a urinary tract infection. The dipstick findings
afebrile. Physical examination reveals suprapubic
support this diagnosis. The most likely causative organism is
tenderness. A urine dipstick reveals ++ nitrites, ++ leukocyte
Escherichia coli. Other possible causative organisms include
esterase and + blood.
Enterococcus and Proteus mirabillis.
What gram stain result would be consistent with the most
M

likely causative organism?

A patient presents with ankle swelling. They are


Minimal change disease
subsequently found to have 4g of protein loss on 24 hours
AI

urinary collection, hypoalbuminaemia and hyperlipidaemia.


Explanation: The patient’s protein loss in their 24 hours urine
A renal biopsy is performed which shows:
collection, hypoalbuminaemia and hyperlipidaemia confirm the
diagnosis of nephrotic syndrome. Causes of nephrotic syndrome
Light microscopy: normal.
include minimal change disease, focal segmental
Immunofluorescence: negative.
glomerulosclerosis, membranous nephropathy and diabetic
Electron Microscopy: podocyte effacement.
©

nephropathy. The findings on renal biopsy are consistent with


minimal change disease.
What is the most likely diagnosis?

4 © AIMTED 2017. All Rights Reserved. See website for disclaimer.


http://www.aimted.com

A 53-year-old female presents to the GP with dizziness that


started 12 hours ago. She has found that when she looks up
Epley manoeuvre
she becomes very dizzy and the room looks as if it is
moving. This happened several times today. Each time it
Explanation: The presence of brief, reproducible episodes of
occurs the sensation that the room is spinning lasts
vertigo with certain head movements strongly suggesting the
approximately 10 seconds. She has been nauseas but has not
diagnosis of benign paroxysmal positional vertigo (BPPV). The
vomited. The patient has no significant past medical history.
positive Dix-Hallpike manoeuvre is pathognomonic for BPPV.
Vital signs are stable and 37.0˚C. Physical examination
BPPV may be treated with a particle repositioning manoeuvre
reveals nystagmus with a fast phase to the left on Dix-
(such as the Epley manoeuvre), or the condition may be observed
Hallpike manoeuvre.
as cases will also typically resolve spontaneously.
What is best management for this patient’s symptoms?
A 25-year-old male presents to the GP with headaches for
the last 3 days. The headaches are located behind his left

17
eye, are stabbing in nature and very severe. With the 100% supplemental oxygen
headaches he gets unilateral tearing and conjunctival
injection of his left eye and the left side of his nose runs. Explanation: This patient has presented with a classical history of
Each headache lasts about 35 minutes and he has had 3 per cluster headaches including severe stabbing retro-orbital,
day over the past 3 days. During the consultation he reports unilateral headaches (with unilateral tearing and conjunctival
that he currently has one such headache beginning and is in injection) of short duration and occurring several times per day

20
severe pain.The patient has no significant past medical over the course of several days. Abortive treatment for a cluster
history. The patient has never smoked and doesn’t drink headache is 100% supplemental oxygen. If this does not work in
alcohol. Vital signs are HR 92, BP 132/76, RR 18, O2 sats 15 minutes then sumatriptan may be used. Verapamil may be
99% RA and 37.1˚C. used prevent further attacks.

What is best treatment alleviate his current headache?


A 67-year-old female presents to the GP with vaginal itch
D
that started insidiously over the past few months and Vulvar biopsy
superficial dyspareunia over the past week. She has had no
vaginal discharge. The patient has a past medical history of Explanation: Lichen sclerosis is an idiopathic inflammatory skin
TE
obesity and type 2 diabetes mellitus, which she manages condition that may occur in multiple demographic groups,
with metformin. She underwent menopause at the age of 54. including postmenopausal women. It is characterised by a white
She is sexually active with her husband and uses no form of plaque (papules and follicular plugs in early stage disease) that
contraception. The patient has a 25 pack-year smoking may become hyperkeratotic, ulcerated, excoriated or have
history and consumes 2 standard drinks of alcohol per day. ecchymoses. The plaque is often pruritic and may bleed due to
She is haemodynamically stable and afebrile. Physical skin fragility. Ulceration or excoriations may be painful. When it
examination reveals a white patch on the labia majora, is more advanced it may cause destruction of the architecture of
M

approximately 1 cm in diameter. The patch has several the perineum with narrowing of the introitus and loss of the labia.
follicular plugs surrounding it. The patch is excoriated. This causes dyspareunia. A definitive diagnosis is reached with a
Otherwise physical examination is non-contributory. What is vulvar biopsy.
the following is the most appropriate investigation?
AI

A 41-year-old male presents to the GP because he has found


that he gets excessively short of breath when trying to jog.
Serum alpha-1 antitrypsin levels
He has been noticing this for the past month. He also has a
chronic cough that is productive of yellow sputum. The
Explanation: The presence of an obstructive pattern on spirometry
patient has no significant past medical history and has never
(with little reversibility), hyperexpanded chest, chronic productive
smoked. Vital signs are stable and 37.1˚C. Physical
©

cough in the setting of shortness of breath on exertion is


examination reveals a barrel chest with a hyperresonant
suggestive of chronic obstructive pulmonary disease (COPD).
percussion note and decreased breath sounds. A CXR
However, this patient has no smoking history. Alpha-1 antitrypsin
demonstrates a hyper-expanded chest. Spirometry reveals an
deficiency may lead to a COPD picture in a younger individual
obstructive pattern with little reversibility. LFTs reveal
who has never smoked. The presence of elevated LFTs provides
elevated ALT and AST.
further support for this diagnosis.
What is the most appropriate diagnostic test?

5 © AIMTED 2017. All Rights Reserved. See website for disclaimer.


http://www.aimted.com

A 63-year-old male presents to the ED with headaches and


increasing confusion for the past 6 hours. He has also
Serum carboxyhaemoglobin
complained of feeling nauseas and dizzy. His wife has
brought him in and reports that he has been fine up until
Explanation: The presence of these vague symptoms with
today. He has been working on the engine of a car he is re-
evidence of appendicular ataxia and a possible predisposing event
building. He has no significant past medical history. Vital
make carbon monoxide poisoning a likely diagnosis. Poorly
signs are HR 106, BP 122/80, RR 18, O2 sats 100% RA and
running engines in enclosed spaces may lead to carbon monoxide
37.0˚C. Physical examination reveals confusion and a
poisoning. Elevated carboxyhaemoglobin levels are diagnostic for
bilateral intention tremor.
carbon monoxide poisoning.
What is the most appropriate investigation?
A 70 year-old with COPD presents to his GP complaining of
increasing shortness of breath. He was previously managing
his symptoms only with salbutamol but this is no longer

17
Salmeterol
providing adequate relief. He has only had one infective
exacerbation in the last 24 months. The patient has a past
Explanation: This patient has COPD and his symptoms are no
medical history of hypertension, which is managed with
longer being managed adequately with salbutamol alone.
enalapril. The patient has a 50 pack-year smoking history
According to current guidelines he should now be offered a long-
but ceased smoking 2 years ago and consumes 2 standard
acting bronchodilator, such as salmeterol or tiotropium. The

20
drinks of alcohol per day.
COPDX guidelines may be referred to for further information.
What treatment may be recommended to this patient at this
time?
A 62-year-old female presents to a GP with a neck lump.
She reports that the lump has grown visibly over the last 2
weeks. She is having difficulty swallowing food and her
D
Anaplastic thyroid carcinoma
voice has become hoarse. Examination reveals a stony hard
nodule that is fixed to underlying structures. Subsequent
Explanation: The diagnosis of anaplastic thyroid carcinoma is
neck ultrasound and fine needle aspiration reveals the lump
supported by the rapid onset and progression of the tumour and its
TE
to be due to thyroid cancer.
degree of invasiveness.
What type of thyroid cancer does the patient most likely
have?
A 53-year-old female presents to the ED with abdominal
pain that started 1 hour ago. The pain is dull and constant
M

and located in the epigastric region. She just finished a large


Biliary colic
meal 2 hours ago that consisted of fish and chips. She has
tried taking antacids and these provided no relief. The
Explanation: This patient has presented with epigastric pain that
patient has had similar pain previously following fatty meals
was precipitated by a fatty meal and not alleviated by antacids.
AI

and it generally resolved within 30 minutes. The patient is


She has had similar episodes previously. The lack of fever and
haemodynamically stable and afebrile. Physical examination
nature of pain helps to distinguish biliary colic from cholecystitis.
reveals no abnormalities.

What is the most likely diagnosis?


A 61-year-old male presents to the ED with chest
©

pain that started 1 hour ago. The patient commenced


Oral-contrast oesophagography
vomiting while drinking with friends after going out for
pizza. No blood was seen in the vomit. Shortly after he
Explanation: The presence of sudden onset chest pain following
commenced vomiting he developed the chest pain, which
vomiting, with a left-sided pleural effusion and subcutaneous
was sudden in onset and worsened by swallowing. The
emphysema makes oesophageal rupture (Boerhaave syndrome)
patient has a 35 pack-year smoking history and usually
the most likely diagnosis. While chest x-ray findings such as a
consumes 3 standard drinks of alcohol per day. He is
pneumomediastinum or unilateral pleural effusion may be
tachycardic and afebrile. A chest x-ray is performed which
strongly suggestive of oesophageal rupture the diagnosis is
demonstrates a left pleural effusion and subcutaneous
confirmed by oesophagraphy (with a water-soluble contrast) or a
emphysema.
chest CT.
What is the most appropriate diagnostic investigation?

6 © AIMTED 2017. All Rights Reserved. See website for disclaimer.


http://www.aimted.com

A 63-year-old female presents to a GP because of diarrhoea


Urinary 5-HIAA
for the past few weeks. The diarrhoea occurs 5-10 times per
day and is non-bloody. She has also been experiencing
Explanation: The combination of diarrhoea, facial flushing and
episodic facial and neck flushing and periodic abdominal
abdominal pain is suggestive of carcinoid syndrome. Carcinoid
pain. She is 5 years post-menopausal and has no ongoing
syndrome is a term used to describe a series of features common
menopause symptoms. There are no apparent triggers for
to carcinoid tumours, which secrete mediators such as serotonin,
these episodes. The patient has no significant past medical
kallikrein and prostaglandins. These tumours may occur in the GI
history. Vital signs are stable and 37.1˚C. Physical
tract or respiratory tract. Urinary 5-HIAA levels may be elevated
examination reveals a diffuse wheeze.
in carcinoid syndrome due to the metabolism of the excess
serotonin.
What is the most appropriate next investigation?
A 67-year-old male presents to the GP with jaundice. His
wife first pointed it out to him 1 week ago. He has had no
pain with the jaundice but has had 10kg of unintentional

17
weight loss over the past 3 months and been feeling
fatigued. He has had no swelling of his peripheries or
Pancreatic cancer
abdomen. He has no significant past medical history. He has
a 40 pack-year smoking history, consumes 2 standard drinks
Explanation: In this case the presence of painless jaundice with
of alcohol per day and has never used recreational drugs. He
weight loss and Courvoisier’s sign makes pancreatic cancer the

20
has no tattoos. He is haemodynamically stable and afebrile.
most likely diagnosis.
Physical examination confirms the presence of jaundice and
reveals a non-tender palpable mass under the costal cartilage
of the right 9th rib.

What is the most likely diagnosis?


A 44-year-old male presents to the GP with epigastric pain,
D
Fasting serum gastrin
diarrhoea and tiredness that started over the last 6-8 weeks.
He has also been feeling short of breath on exertion. The
Explanation: This patient’s endoscopy has identified a duodenal
patient has a past medical history of recurrent peptic ulcers,
ulcer. Almost all duodenal ulcers are due to Helicobacter pylori
which have been treated with Helicobacter pylori eradication
TE

infection. The lack of Helicobacter pylori in this instance, and his


therapy. Vital signs are stable and 37.0˚C. Physical
history of recurrent peptic ulcers is suspicious for an underlying
examination reveals pallor of the palmar creases. CBE and
cause, such as Zollinger-Ellison syndrome. Fasting serum gastrin
iron studies reveal iron deficiency anaemia. An endoscopy
may support the presence of Zollinger-Ellison syndrome, in
identifies a bleeding duodenal ulcer and tests for
conjunction with abdominal imaging (such as a CT). If fasting
Helicobacter pylori return negative.
gastrin levels are not diagnostic a secretin stimulation test may be
M

performed.
What is the most appropriate investigation?

A 69-year-old female presents to the GP with choking when


Barium swallow
trying to swallow solid food that started 3 months ago. She
AI

also describes the regurgitation of partially eaten food


Explanation: In this case the presence of regurgitation of food
several hours after meals and reports suffering from ‘bad
several hours after eating in conjunction with halitosis makes a
breath’. The patient has no significant past medical history.
pharyngeal pouch (otherwise known as Zenker diverticulum) the
Her vital signs are stable and she is afebrile. Physical
most likely diagnosis. Barium swallow is the diagnostic test of
examination reveals no abnormalities.
choice for a pharyngeal pouch (although endoscopy may later be
required for endoscopic diverticulotomy as a treatment modality).
©

What is the most appropriate investigation at this stage?

7 © AIMTED 2017. All Rights Reserved. See website for disclaimer.


http://www.aimted.com

A 70-year-old female presents to the ED with sudden onset


vision loss in her right eye that started 1 hour ago. She was
at rest when over the course of several seconds the vision in
her right eye became significantly worse. The patient has a
past medical history of hypertension, diabetes mellitus type Central retinal vein occlusion
2 and dyslipidaemia, which is managed with enalapril,
amlodipine, metformin and atorvastatin. The patient has a 55 Explanation: In this case the sudden onset painless unilateral
pack-year smoking history and consumes 3 standard drinks vision loss with significant retinal haemorrhages in an individual
of alcohol per day. The patient’s vital signs are stable and with numerous risk factors including hypertension, diabetes
she is afebrile. Physical examination reveals visual acuity of mellitus and dyslipidaemia makes central retinal vein occlusion
6/6 in the left eye and 6/48 in the right eye. She has a right- the most likely diagnosis.
sided relative afferent pupillary defect. Ophthalmoscopy
severe retinal haemorrhages throughout the fundus.

17
What is the most likely diagnosis?

A 23-year-old male presents to the ED with testicular


swelling that started over the past 3-4 hours. He reports that
he was playing soccer prior to the onset of the swelling, Haematocele
during which time the ball was kicked into his groin. This

20
caused significant pain. The patient has no significant past Explanation: The history of trauma, with a painful swelling that
medical history. Vital signs are stable and 37.1˚C. Physical the testicle cannot be palpated separately from supports the
examination reveals a tender diffusely swollen right side of presence of a haematocele. The swelling not transilluminating
the scrotum. The right testicle is not palpable separate from supports this diagnosis, although is a non-specific finding (for
this swelling. The swelling does not transilluminate. example, inguinal hernias do not transilluminate).

What is the most likely diagnosis?


D
A 64 year-old male presents to the emergency department
Ureteric stent
with loin pain. He has a past medical history of high
cholesterol. The patient is haemodynamically stable and has
TE
Explanation: The diagnosis of nephrolithiasis has been confirmed
a temperature of 38.7˚C. A left-sided kidney stone is
by non-contrast abdominal CT. The patient’s fever is suggestive
identified on non-contrast abdo CT. The stone is 5mm in
of an upper urinary tract infection in conjunction with the stone.
size. The patient has 2 functioning kidneys and eGFR >90.
This is an indication for a ureteric stent or nephrostomy. Given
the patient is of an acceptable anaesthetic risk the most
What is the most appropriate definitive treatment for this
appropriate treatment is a ureteric stent.
patient?
M

A 67-year-old male presents to the GP with blood in his


urine for the past 6-7 days. The blood is visible in his urine
throughout the entire stream (present at beginning, middle
Abdominal CT
and end of urination). He has also had loin pain for the past
AI

month. The patient has no significant past medical history.


Explanation: This patient has presented with the classical triad of
The patient has a 50 pack-year smoking history and
renal cancer: haematuria, loin pain and flank mass. This is
consumes 3 standard drinks of alcohol per day. Vital signs
sometimes referred to as the ‘too late triad’ as it is associated with
are stable and 37.1˚C. Physical examination reveals a left
a poor prognosis.
sided flank mass that moves inferiorly with inspiration.
©

What is the most appropriate investigation?

8 © AIMTED 2017. All Rights Reserved. See website for disclaimer.


http://www.aimted.com

A 63-year-old female presents to the ED with chest pain and


back pain that started 1 hour ago. The pain was sudden in
IV labetalol
onset and is severe. She describes the pain as ripping. The
patient has a past medical history of hypertension and
Explanation: In this case the sudden onset ripping back pain in
dyslipidaemia, which is unmanaged. The patient has a 45
conjunction with a >20mmHg difference in arm blood pressures,
pack-year smoking history and consumes 2 standard drinks
a pulse deficit and numerous cardiovascular risk factors makes
of alcohol per day. Vital signs are HR 122, BP 154/90 on the
aortic dissection the most likely diagnosis. This is further
right arm (left arm 126/88), RR 18, O2 sats 100% RA and
supported by the chest x-ray findings. IV labetalol will decrease
37.2˚C. Physical examination reveals an absent left radial
blood pressure and heart rate to try and prevent further damage.
pulse. A chest x-ray demonstrates a widened mediastinum.
Urgent vascular surgery may also be required although further
The patient is admitted to ICU and commenced on morphine
investigation (such as CT angiography) results should be gained
for analgesia.
prior to this if possible. Thrombolytics should not be used.
What is the most appropriate treatment at this stage?

17
A 32-year-old female presents to the GP with a breast lump
that she identified on self-examination yesterday. She has Fat necrosis
never had a breast lump before. She had trauma to the left
breast while playing soccer recently. Physical examination Explanation: In this case the present lump has features suggesting
reveals a lump in the lower outer quadrant of the left breast. malignancy (hard, irregular borders). However, fat necrosis may

20
The lump is hard with irregular borders. It is non-tender. mimic malignancy and the history of trauma to the breast in
There is no palpable axillary lymphadenopathy. question increases the likelihood of this diagnosis. However,
further investigation would still be required.
What is the most likely diagnosis?

Urinary metanephrines
A 49-year-old female presents to the GP with headaches that
D
started 3 weeks ago. She has periodic headaches that are
Explanation: The presentation with episodic headache,
becoming more frequent, lasting approximately 1 hour each.
palpitations and diaphoresis with background hypertension makes
They are now occurring once every 24-48 hours. During
pheochromocytoma the most likely diagnosis.
these headaches she has palpitations and sweats excessively.
TE
Phaeochromocytoma is caused by an abnormal proliferation of
Her last headache was 12 hours ago. The patient has no
chromaffin cells, which are normally present in the adrenal
significant past medical history. The patient has never
medulla and produce catecholamines (adrenaline and
smoked, doesn’t drink alcohol and doesn’t use recreational
noradrenaline). Metanephrines are breakdown products of
drugs. HR 106, BP 160/92 and temperature 37.1˚C. Physical
catecholamines and therefore elevated levels support the
examination reveals no abnormalities.
diagnosis of pheochromocytoma. Metanephrines may be
measured in the urine or the plasma, however a 24 hour urinary
M

What is the most appropriate investigation?


collection is more sensitive.

A 60-year-old male presents to the GP with difficulty


Achalasia
swallowing that started 2 months ago. He has difficulty
AI

swallowing liquids and solids. A barium swallow reveals a


Explanation: A bird’s beak appearance on barium swallow is
bird’s beak appearance.
strongly suggestive of achalasia. However, the diagnostic gold
standard is manometry.
What is the most likely diagnosis?
A 47-year-old female presents to the ED with abdominal
pain that started 6 hours ago. The pain is located in the right
©

upper quadrant, is constant and is sharp in nature.


Abdominal ultrasound
Previously, over the past few weeks, she has had abdominal
pain intermittently after meals. This pain was dull in nature
Explanation: This patient has RUQ pain and fever in the setting of
and in the epigastric region. The pain today started after a
likely previous biliary colic. She is not jaundiced. Biliary colic is
large meal that included a sizeable portion of fast food
not associated with a fever. Ascending cholangitis is associated
burgers. Her periods are regular every 28 days. Vitals are
with jaundice and more systemic upset than this patient is
stable and she has a temperature of 38.0˚C. Physical
demonstrating. The diagnostic investigation for acute
examination reveals right upper quadrant tenderness. She is
cholecystitis is an abdominal ultrasound.
not jaundiced.

What is the most appropriate diagnostic investigation?

9 © AIMTED 2017. All Rights Reserved. See website for disclaimer.


http://www.aimted.com

A 33-year-old male presents to the ED with abdominal pain


that started 20 hours ago. The pain is severe, constant and
Serum lipase
located in the epigastric region and left hypochondrium. The
pain radiates to the middle of his back. He has never had
Explanation: The character and location of the pain in conjunction
pain like this before. He has no significant past medical
with the deranged vital signs and recent binge drinking makes
history. He does not smoke and consumes an average of 3
acute pancreatitis the most likely diagnosis in this patient. An
standard drinks of alcohol per day. He has just returned from
elevated serum lipase may confirm the diagnosis of acute
his brother’s wedding at which he consumed 15 standard
pancreatitis. The most likely cause of this patient’s acute
drinks of alcohol per day for several days. The patient is
pancreatitis is his recent alcohol consumption so endoscopic
tachycardic and has a temperature of 38.3˚C. Physical
retrograde cholangio-pancreatography is likely unnecessary
examination reveals epigastric and left hypochondrium
(assuming and abdominal ultrasound does not show evidence of
tenderness and guarding.
obstructing gallstones).
What is the most appropriate diagnostic investigation?

17
A 44-year-old female presents to the GP with right ear pain
for the last 1 day. The pain began gradually and is now quite
Pseudomonas aeruginosa
severe. She is haemodynamically stable and afebrile.
Physical examination reveals pain on pulling of the right
Explanation: In this case the presence of a visibly oedematous and
pinna. Pain limits examination with the otoscope but the
red external ear canal with pain on pulling of the pinna makes

20
tympanic membrane appears intact. The external ear canal is
otitis externa the most likely diagnosis. Pseudomonas aeruginosa
red and oedematous.
is the most common causative organism of otitis externa.
What is the most likely causative organism?
A 14-year-old male presents to the GP with headaches over
the past 4-6 months. The patient has no significant past
medical history. Vital signs are stable and 36.9˚C. Brain Craniopharyngioma
D
MRI reveals a mass with cystic and solid components in the
suprasellar cistern with evidence of calcifications. The cystic Explanation: The MRI findings are consistent with a
mass enhances when contrast is added. craniopharyngioma.
TE

What is the most likely diagnosis?

A 72-year-old female presents to the ED with a sudden


change of vision in her left eye that started 1 hour ago. She
reports that she was watching TV when suddenly she
realised that she couldn’t see part of the screen with her left
M

AION
eye. Over the past 3 days she has had a left-sided temporal
headache and has had general muscle soreness and malaise.
Explanation: In this case the patient’s clinical features indicate a
The patient has no significant past medical history. Her vital
diagnosis of giant cell arteritis (temporal arteritis). Her sudden
signs are stable and her temperature is 37.8˚C. Physical
AI

altitudinal vision loss in the setting of giant cell arteritis makes


examination reveals loss of the lower half of her visual field
anterior ischaemic optic neuropathy (AION) the most likely
in her left eye, tenderness on palpation of the left temporal
diagnosis.
region and scalp. The visual acuity of the superior half of her
field of vision of her left eye is unaffected.

What is the most likely cause of her loss of vision?


©

A 43-year-old female is brought to ED 6 hours following an Haemodialysis


intentional lithium overdose as a suicide attempt. No other
drugs were used. The patient is somnolent so little history Explanation: In this case the patient has severe lithium toxicity as
can be gained. The patient has a past medical history of indicated by the depressed level of consciousness. Severe lithium
bipolar disorder, which is managed with lithium and toxicity should be treated by haemodialysis (in addition to
escitalopram. Her vital signs are stable and her temperature stopping the administration of lithium). Polyethylene glycol may
is 37.1˚C. Physical examination reveals that the patient has a be used for bowel irrigation in acute lithium toxicity (activated
depressed level of consciousness, has myoclonic jerks and a charcoal does not decrease absorption of lithium) when the
coarse tremor affecting both hands. patient is alert and the overdose occurred <4 hours ago. Another
important step in the treatment of lithium toxicity is ensuring the
What is the most appropriate treatment? patient is adequately hydrated.

10 © AIMTED 2017. All Rights Reserved. See website for disclaimer.


http://www.aimted.com
A 71-year-old female is brought to the GP by her son
because he is concerned about her ability to cope at home.
Tau
He reports that she is increasingly forgetful and is not
maintaining her home as she used to. The patient reports that
Explanation: The combination of memory loss with nominal
she is does lose her keys frequently and keeps forgetting
aphasia and impaired visuospatial function (losing objects) in the
people’s names but does not think that her memory has been
absence of other symptoms/signs makes Alzheimer’s disease the
otherwise impaired. She has a past medical history of
most likely diagnosis. Two of the prevailing theories regarding
hypertension, which is managed with enalapril. Vital signs
the mechanisms underlying Alzheimer’s disease are the amyloid
are stable and 36.9˚C. Physical examination reveals the
hypothesis (overproduction/decreased clearance of beta-amyloid
patient has a dishevelled appearance. She has a mini-mental
à formation of beta-amyloid oligomers à deposition as
state score of 24.
extracellular amyloid plaques à cell death) and tau hypothesis
(altered phosphorylation and accumulation of tau intracellularly
This condition is thought to involve the accumulation of
à formation of intracellular neurofibrillary tangles à cell death).
which substance leading to the formation of intracellular

17
neurofibrillary tangles?
A 59-year-old female is brought to the GP by her husband
because he is concerned that she is behaving oddly. Over the
past several months she has become increasingly rude and Frontotemporal dementia
apathetic. For example, she has begun swearing at the
neighbour’s young children when they visit whereas she was Explanation: The combination of executive dysfunction (e.g.

20
previously always been patient and kind with them. She has inability to plan) and change in personality including apathy,
also been bathing less frequently and stopped making plans irritability and poor self-care in conjunction with frontal release
to see her friends. Physical examination reveals a bilateral signs (bilateral palmar grasp reflex) make frontotemporal
palmar grasp reflex. dementia the most likely diagnosis in this patient.

What is the most likely diagnosis?


D
A 32-year-old female is brought to a psychiatrist by her
friend who is concerned about her. The friend reports that
the patient has a firmly held belief that a famous actor from
TE
a series of superhero movies is in love with her, despite the
Delusional disorder
fact they have never met. The patient is secure in her view
that this is true and cannot be dissuaded. She reports that she
Explanation: This patient has a fixed false belief but has
has known this for several years, since his first movie, but
apparently normal function otherwise. She has no apparent
that he is waiting for the ‘right time’ to contact her. She has
hallucinations or bizarre behaviour. This fits with the description
a job as an accountant and is apparently well otherwise. She
of a delusional disorder. The type of delusion, which she is
has no significant past medical history. She displays an
M

describing could be classified as erotomania.


appropriate range of emotions during the consultation. There
is no evidence that she is responding to internal stimuli.

What is the most likely diagnosis?


AI

A 27-year-old female presents to the GP with a depressed Bipolar disorder type 2


mood for the last 3 weeks. She reports that she is sleeping all
the time and can’t find any enjoyment in anything she does. Explanation: This patient is having a depressive episode
She reports that she was feeling much better about 1 month following what sounds like a hypomanic episode. The previous
ago where, for a few days, she was so energetic that she episode was likely hypomania, rather than mania, because it did
©

finished all her projects at work and cleaned the house at not cause significant social/occupational dysfunction, there were
night. The patient has no significant past medical history and no psychotic features and it did not require hospitalisation. Its
doesn’t smoke, drink alcohol or use recreational drugs. duration was also less than 1 week. Therefore, this combination of
a depressive episode and a hypomanic episode supports the
What is the most likely diagnosis? diagnosis of bipolar disorder type 2.

11 © AIMTED 2017. All Rights Reserved. See website for disclaimer.


http://www.aimted.com
A 37-year-old male presents to the GP with a history of
headaches consistent with tension headaches. He reports that
he spends all day on his computer and the headaches annoy
him because it stops him from doing his job. Throughout the
Schizoid personality disorder
consultation he appears uncomfortable, makes poor eye
contact and has a flat affect. The GP, concerned that he may
Explanation: This patient fulfils the DSM-V criteria for a schizoid
have depression, takes a further history. It is revealed that he
personality disorder. The long-term duration of his lifestyle
lives alone, has no close friends and has no desire for close
supports this diagnosis. He has not actually stated that he has a
friends. This has been the case for the last 17 years since he
depressed mood (although it may be said that he has features of
moved out of home. He reports not enjoying many activities,
anhedonia) and in fact seems reasonably satisfied with his
although he ‘doesn’t mind’ being at work. He denies
lifestyle.
difficulties sleeping, with appetite or energy levels as well as
denying suicidal ideation.

17
What is the most likely diagnosis?

A 25-year-old male presents to the GP with difficulty


sleeping for the past 6-8 months. He reports that he worries Psychological intervention
all the time and that this prevents him from falling asleep.
He also complains of poor concentration as well as feeling Explanation: The patient fulfils the DSM-V diagnostic criteria for

20
tired and irritable most of the time. He does not take any generalised anxiety disorder (GAD). Psychological interventions
medication, smoke, drink or use recreational drugs. Thyroid such as cognitive behavioural therapy or relaxation strategies are
function tests reveal no abnormalities. recommended for GAD. Antidepressants may also be required in
some cases, usually commencing with an SSRI.
What is the appropriate treatment at this stage?
D
A 22-year-old female presents to the GP with amenorrhoea
Anorexia nervosa
for the past 2 months. She has taken several pregnancy tests
and is not pregnant. She reports repeated vomiting after
Explanation: The patient’s low BMI is likely responsible for her
meals. She denies heat or cold intolerance. Physical
TE

amenorrhoea. Given the repeated vomiting following meals in the


examination reveals a BMI of 16, lanugo hair and decreased
setting of this low BMI this presentation is suggestive of anorexia
skin turgor.
nervosa. Further questions should be asked regarding her opinion
of her body image and weight.
What is the most likely diagnosis?

A 42-year-old female presents to the GP with flashbacks and


M

irritability relating to a mugging that occurred 2 weeks ago.


During the mugging she was threatened with a knife but was Acute stress disorder
not harmed beyond contusions sustained during a fall when
the aggressor pushed her over. She has been unable to go Explanation: This patient is demonstrating intrusion symptoms,
AI

back to the part of the city in which the mugging occurred. avoidance symptoms, negative changes in mood/cognition and
She has also noticed that she has an exaggerated startle hyperarousal symptoms. This is following exposure to threatened
response and family members have reported that she seems personal serious injury in regards to the mugging. The duration of
angrier and less happy than usual. The patient has also been the patient’s symptoms (>3 days but less than <1 month) indicate
sleeping poorly. that the best diagnosis at this stage is acute stress disorder.
©

What is the most likely diagnosis?

12 © AIMTED 2017. All Rights Reserved. See website for disclaimer.


http://www.aimted.com
A 27-year-old male is brought to the ED by his wife because
he has had increasing confusion over the last 3 days. The
patient is disoriented in place and time. The patient has a
past medical history of depression, for which he takes
sertraline, and recently been diagnosed schizophrenia for Neuroleptic malignant syndrome
which he has been commenced on chlorpromazine. The
patient has a 10 pack-year smoking history and consumes 4 Explanation: The presence of mental state changes, autonomic
standard drinks of alcohol per day. Vital signs are HR 110, instability, muscle rigidity and a fever with an elevated creatine
BP 172/106, RR 20, O2 sats 100% RA and 38.6˚C. Physical kinase in a patient recently commenced on a typical antipsychotic
examination reveals generalised muscle rigidity, decreased makes neuroleptic malignant syndrome the most likely diagnosis
deep tendon reflexes and normal sized pupils that are equal in this patient.
and reactive to light. Otherwise physical examination is non-
contributory. Investigations reveal elevated creatine kinase.

17
What is the most likely diagnosis?

A 21-year-old female is brought to the ED by her parents


following a deliberate drug overdose 1 hour ago. The patient ECG
took an unknown quantity of her mother’s amitriptyline,
which she has for chronic pain. The patient reports a Explanation: Tricyclic antidepressant overdoses may cause

20
headache, dry mouth and nausea. She is not drowsy. The several ECG abnormalities, including QT prolongation and QRS
patient is tachycardic and has a temperature 37.1˚C. widening. Such ECG changes guide management with sodium
bicarbonate.
What is the most important investigation at this stage?

A 69-year-old male is brought to the GP by his wife because


she is worried that he has been behaving oddly. Over the
D
past 4-6 months he has been having increasing difficulty
making plans and keeps losing objects. He has also become
prone to having multiple ‘naps’ during the day at odd times.
Dementia with Lewy bodies
TE

He has now started reporting seeing birds in the house,


which his wife has not seen. She also reports he’s been
Explanation: The combination of executive and visuospatial
walking more slowly for the past few months and seems
dysfunction, fluctuations in conscious state, visual hallucinations
constantly on the verge of falling. The patient has a past
and parkinsonism makes Dementia with Lewy bodies the most
medical history of dyslipidaemia, which is managed with
likely diagnosis.
atorvastatin. The patient has never smoked and doesn’t drink
M

alcohol. Vital signs are stable and 37.0˚C. Physical


examination reveals bradykinesia and a shuffling gait.

What is the most likely diagnosis?


AI

A 71-year-old male is brought to the GP by his son because


of memory and behaviour changes. The son reports that over
the past couple of years on several occasions he has noticed
a step-by-step decline in the patient’s level of function. The
Brain MRI
patient is now more apathetic, struggles to make plans and
has poor compliance with his medications. The patient has a
Explanation: In this patient with numerous atherosclerotic risk
©

past medical history of hypertension, dyslipidaemia and


factors there is a history of a stepwise decline in cognitive
stable angina, which is managed with hydrochlorothiazide,
function making vascular dementia the most likely diagnosis. A
ramipril, atorvastatin and GTN. The patient has a 40 pack-
brain MRI is the imaging modality of choice to demonstrate
year smoking history and consumes 2 standard drinks of
previous infarcts and confirm the diagnosis. Despite this, it is still
alcohol per day. Vital signs are stable and 37.0˚C. Physical
important to investigate possible reversible causes of dementia.
examination reveals no abnormalities.

Which of the following investigations is most likely to be


abnormal?

13 © AIMTED 2017. All Rights Reserved. See website for disclaimer.


http://www.aimted.com
A 28-year-old female is brought to the GP by her husband.
For the past 3 days he has noticed that she has had a very Schizoaffective disorder
depressed mood and apparently been responding to voices
that he cannot hear. The patient reports hearing warnings Explanation: The patient has had an episode with depressed mood
about the end of the world and that she is the only one who with anhedonia concurrently with the onset of her psychotic
can stop it. She also reports not enjoying any of her hobbies symptoms. These psychotic symptoms have then persisted
anymore. The patient has no significant past medical history following the resolution of the mood symptoms. This timeline
and doesn’t smoke, drink alcohol or use recreational drugs. fulfils the criteria for schizoaffective disorder. If the depressed
Vital signs are stable and 37.0˚C. Over the course of the next mood had preceded the psychotic symptoms then depression with
few months her mood symptoms resolve but her delusions psychotic feature would be more likely. If the psychotic features
and auditory hallucinations remain. had started first then schizophrenia with concurrent depression
would be more likely.
What is the most likely diagnosis?
A 25-year-old female presents to the ED with chest pain that

17
started 20 minutes ago. The chest pain is centrally located.
In the last 20 minutes she has also had palpitations, felt short
of breath, dizzy and nauseas. She feels as if she is ‘about to
die’. She also reports tingling around her mouth and in her Panic attack
hands. The patient has a past medical history of depression,
which is managed with sertraline. Vital signs are HR 120, Explanation: In this instance, further history regarding what was

20
BP 124/80, RR 26, O2 sats 100% RA and 37.0˚C. Physical happening at the time that may have precipitated this panic attack
examination reveals diaphoresis. ECG reveals no would be helpful. As can be seen here it may be challenging to
abnormalities aside from sinus tachycardia. The symptoms conclusively distinguish between a panic attack and problems
resolve of their own accord after another 15 minutes. A D- such as a pulmonary embolus without investigations.
dimer is within the normal range and so are her troponin
levels at 6 hours.
D
What is the most likely diagnosis?
A 24-year-old female presents to the GP with concerns
regarding a rash on the back of her right hand. The rash
TE

appears callused and slightly excoriated. Further history Bulimia nervosa


reveals that the patient induces repeated bouts of vomiting
following most meals, particularly if it is a large meal during Explanation: The normal range BMI in conjunction with the
which she feels out of control of the amount she eats. This binge-eating and purging support the diagnosis of bulimia
occurs several times per week. These actions are driven by nervosa. The ‘rash’ on the patient’s hand is otherwise known as
recurrent concerns that she is fat. Physical examination Russel’s sign and occurs with repeated manually self-induced
M

reveals a BMI of 21 and tooth enamel erosion. vomiting.

What is the most likely diagnosis?


A 5-year-old child presents to the GP with his parents
AI

reporting his teachers have been complaining about his


Attention deficit hyperactivity disorder
behaviour at school. Both at school, at home and at his
grandparents house he has been observed to be forgetful,
Explanation: This patient is demonstrating evidence of
lose objects and avoid activities which require sustained
inattention, impulsivity and hyperactivity in multiple settings. He
mental effort. He is also described as very energetic and
meets the DSM-V criteria for attention deficit hyperactivity
often stands and runs in class as well as interrupting other
©

disorder. There is no evidence to suggest that he has autism such


classmates. His parents report that he doesn’t snore and they
as limited social verbal or non-verbal reciprocity, fixed interests
have never noticed any pauses in his breathing during the
or adhering to fixed schedules. His lack of snoring or pauses in
night. The patient has no significant past medical history. He
breathing decreases the likelihood of obstructive sleep apnoea as
is haemodynamically stable and afebrile and physical
does his energetic state during the day. Oppositional defiant
examination reveals no abnormalities. He is on the 60th
disorder typically manifests with more deliberate disobedience
percentile for both weight and height and has not crossed
and arguing. The normal physical examination decreases the
any major percentile lines.
likelihood of otitis media with effusion.
What is the most likely diagnosis?

14 © AIMTED 2017. All Rights Reserved. See website for disclaimer.


http://www.aimted.com
A 26-year-old female is brought to the GP by her boyfriend
because he is worried about how often she checks the locks
Obsessive compulsive disorder
in her house. The patient checks the locks on the doors and
windows of her house 5-10 times each time she leaves the
Explanation: This patient is describing an obsession (the
house. This is impairing her daily functioning due to the
unwanted, intrusive, recurrent thought regarding harm to herself
time it takes to leave the house. She reports that she checks
or housemates) and a compulsion (checking the locks recurrently)
the locks this often because she has the worrying feeling that
to reduce the anxiety associated with the obsession. These
if she doesn’t check them ‘something bad’ will happen to
obsessions are ego-dystonic (unwanted) as opposed to the
her or her housemates. She doesn’t wish to have this feeling
thoughts and actions in obsessive compulsive personality
but cannot help it recurrently coming into her head.
disorder, which are ego-syntonic.
What is the most likely diagnosis?

A 35-year-old male presents to the GP because of pain and

17
weakness in his right arm since yesterday. Yesterday while
helping a friend move house when someone dropped a box
of belongings, which he caught, but at the same time heard a
Biceps tendon rupture
‘pop’ and then experienced pain and weakness in the right
arm. Vitals are stable and he is afebrile. Physical
Explanation: The presence of sudden onset pain, bruising and a

20
examination reveals asymmetrical appearance to the right
‘popeye’ sign (the asymmetrical biceps bulging) make a biceps
and left biceps brachii with the right bicep appearing to have
tendon rupture the most likely diagnosis is this patient.
a bulge closer on the anterior aspect of the arm closer to the
elbow. There is also bruising over the right bicep.

What is the most likely diagnosis?


D
A 45-year-old female presents to the GP with pain and
tingling in her right hand. She notices that it is worst when
Median nerve
she is typing at work and during the night. Physical
examination reveals atrophy of the thenar eminence,
TE

Explanation: This patient has presented with the classical features


weakness of thumb opposition and decreased sensation to
of carpal tunnel syndrome. Carpal tunnel syndrome is caused by
light touch over the lateral three digits.
compression of the median nerve in the carpal tunnel.
Which nerve is most likely to have been injured?
Galeazzi fracture
M

Explanation: The fracture and mechanism of injury described is a


A 70-year-old female presents to ED with right hand and
classic Galeazzi fracture. A Smith fracture is a distal radius
wrist pain following a fall onto and outstretched hand with a
fracture with volar angulation. A Colles fracture is a fracture of
flexed elbow. An x-ray reveals a distal radius fracture with
AI

the distal radius with dorsal and radial angulation (often described
dislocation of the distal radiounlar joint
as a ‘dinnerfork’ deformity). A Boxer fracture is a 5th metacarpal
fracture typically occurring, as the name suggests, from the
What is the eponymous name for this type of fracture?
patient punching another person or object. A Monteggia fracture
is a transverse fracture of the proximal ulna and dislocation of the
radial head.
©

A 40-year-old male presents to the GP with pain in his right


Guyon’s canal syndrome
hand when he rides his bike. He reports that when he hits a
bump in the road he gets shooting pain from his right hand
Explanation: This patient has presented with pain and
and wrist. He has been ignoring it until now but has recently
paresethesia in the ulnar nerve distribution. Ulnar nerve
found he has paraesthesia in the medial one and half digits
symptoms in conjunction with bike riding may be called
of his right hand after bike rides.
‘handlebar palsy’ and is typically caused by compression of the
ulnar nerve in Guyon’s canal against the handlebars.
What is the most likely diagnosis?

15 © AIMTED 2017. All Rights Reserved. See website for disclaimer.


http://www.aimted.com
A 65-year-old male presents to the GP with fatigue
and muscle weakness that started several months ago. He
feels tired all the time and finds the weakness particularly
Gastric cancer
challenging when standing from a chair or climbing stairs.
He has also lost 7kg in weight over the past 2 months
Explanation: This patient has presented with features consistent
unintentionally. The patient has no significant past medical
with dermatomyositis including proximal muscle weakness,
history. Vital signs are stable and 37.0˚C. Physical
heliotrope rash and Gottron’s papules. Dermatomyositis may
examination reveals 4/5 proximal muscle weakness of the
occur as a paraneoplastic phenomenon, which is supported by the
arms and legs. In addition a violaceous macular
presence of recent weight loss in this patient. Cancers which
discolouration is seen on the superior eyelids. Flat plaques
dermatomyositis has been associated with include lung cancer,
are present on the dorsal aspect of the patient’s
breast cancer and gastric cancer.
metacarpophalangeal joints bilaterally.

What is the most likely diagnosis?

17
A 23-year-old male presents to the GP with joint pain that
started 1 week ago. He has pain in his right knee right hip
and left shoulder. This pain is worst in the morning and Enteropathic arthritis
associated with 90 minutes of stiffness when he first wakes
up. He has also had diarrhoea and intermittent abdominal Explanation: The history of diarrhoea is more consistent with
pain for the last 3 weeks, with some bowel motions inflammatory bowel disease than an infection (such as

20
containing red blood. The patient has a past medical history campylobacter, which may lead to reactive arthritis, or
of schizophrenia, which is managed with olanzapine. He adenovirus) due to the duration of its symptoms. The combination
lives in Hobart and has not travelled interstate or overseas of possible inflammatory bowel disease and asymmetrical
recently. Vital signs are stable and 37.3˚C. Physical inflammatory (indicated by morning stiffness) oligoarticular joint
examination reveals his affected joints and swollen and pain makes enteropathic arthritis the most likely diagnosis in this
tender with a complete range of passive motion. patient.
D
What is the most likely diagnosis?
A 27-year-old male presents to the ED because he has been
coughing up blood for the past 4 hours. He has been
TE

coughing up streaks of blood and had minimal chest pain.


Goodpasture’s syndrome
He has never had anything like this before. The patient has
no significant past medical. Vital signs are HR 84, BP
Explanation: The presence of anti-GBM antibodies in conjunction
170/92, RR 22, O2 sats 98% RA and 37.1˚C. Physical
with haemoptysis and likely renal dysfunction make
examination reveals bilateral coarse crackles over the lung
Goodpasture’s syndrome the most likely diagnosis. Early
bases. Urine dipstick reveals +++ blood and ++ protein.
treatment with a combination of plasmapheresis and
M

Serology is positive for anti-glomerular basement membrane


corticosteroids is vital for long-term prognosis.
antibodies.

What is the most likely diagnosis?


AI

A 57-year-old female presents to the ED due to left knee Intra-articular steroids


pain of 3 hours in duration. Physical examination reveals a
swollen erythematous left knee. The patient has a past Explanation: The joint aspiration results are diagnostic for gout.
medical history of type 2 diabetes mellitus and chronic The current guidelines recommend NSAIDs (such as
kidney disease (eGFR 48), which is managed with insulin. indomethacin) as first line treatment for gout. However, in this
Joint aspiration is performed and reveals negatively case the patient’s chronic kidney disease is a contraindication to
©

birefringent crystals with a needle shape. NSAIDs. Therefore, intra-articular steroids are appropriate.
Allopurinol and probenecid are used to prevent gout attacks and
What is the most appropriate treatment at this stage? should not be commenced during an acute attack.

16 © AIMTED 2017. All Rights Reserved. See website for disclaimer.


http://www.aimted.com
A 32-year-old female presents to the ED with chest pain that
started 1 hour ago. The pain is sharp in nature and worse NSAIDs
when lying down. The patient has a past medical history of
systemic lupus erythematosus, for which she currently Explanation: The nature of this patient’s pain and ECG findings
requires no medication. Vital signs are stable and 37.3˚C. are consistent with pericarditis. This may be due to her lupus.
Physical examination reveals no abnormalities. An ECG First line treatment for lupus pericarditis is NSAIDs. If these do
reveals widespread up-sloping ST elevation. not provide adequate symptomatic relief then corticosteroids may
be used.
What is the most appropriate initial treatment?

A 62-year-old male presents to his GP because of stiffness in


his right shoulder for the past week. It is stiff in when he
tries to move it in any direction. He has minimal pain but
Adhesive capsulitis

17
reports that it was sore to move in every direction for 2-3
months while during onset of this stiffness. Vitals are stable
Explanation: The presence of a history of shoulder pain, that is
and he is afebrile. Physical examination reveals limited
now resolving, followed by a significant limitation to both active
active and passive range of motion in all directions. There is
and passive range of motion in the absence of joint crepitus, or
a particular lack of external rotation. There is no joint
other symptoms, makes adhesive capsulitis (otherwise known as a
warmth or crepitus. The left shoulder demonstrates no

20
frozen shoulder) the most likely diagnosis in this patient.
abnormalities.

What is the most likely diagnosis?

A 29-year-old male presents to the GP because of pain in his


right shoulder for the past 2 weeks. He works as a manual
Bicipital tendonitis
labourer and this pain is impairing his work. The pain is ‘at
D
the front’ of his shoulder and is worst when he is carrying
Explanation: The presence of pain in the region of the bicipital
heavy objects. Physical examination reveals pain on
groove (anterior shoulder several centimetres inferior to
palpation of the anterior aspect of the shoulder several
acromion) along with a positive Yergason test (pain is reproduced
TE
centimetres inferior to the acromion. This pain is reproduced
with supination against resistance) and Speed test (anterior
with supination against resistance. Anterior shoulder pain
shoulder pain reproduced with shoulder flexion, with an extended
also occurs with shoulder flexion with an extended elbow
elbow, and resistance applied to the forearm) make bicipital
and resistance applied to the forearm.
tendonitis the most likely diagnosis.
What is the most likely diagnosis?
M

A 22-year-old male presents to the GP with pain in his right


hand and wrist. He reports that the pain is worst when using
his smartphone. Physical examination reveals tenderness De Quervain’s tenosynovitis
AI

over the radial styloid. When the GP holds the patient’s right
thumb and then sharply pulls it to cause ulnar deviation of Explanation: The test performed by the GP is Finkelstein’s test. It
the hand the patient reports pain along the radial side of the indicates that De Quervain’s tenosynovitis is the most likely
wrist. diagnosis.

What is the most likely diagnosis?


©

A 65-year-old male presents to the GP with pain in his


hands. Physical examination reveals restricted range of Paracetamol
motion and bony swellings of the distal interphalangeal
joints of several fingers. There is squaring of the first Explanation: The presence of Heberden’s nodes and squaring of
carpometacarpal joint on both hands. There is no erythema the first carpometacarpal joint makes osteoarthritis the most likely
or soft tissue swelling over the affected joints. diagnosis. First line treatment is with paracetamol. NSAIDs may
be added if paracetamol does not provide adequate pain relief.
What is the most appropriate treatment?

17 © AIMTED 2017. All Rights Reserved. See website for disclaimer.


http://www.aimted.com

A 59-year-old female presents to the GP because of pain in


her right shoulder for the past week. She has been
renovating her house and doing the painting herself but the Rotator cuff impingement
shoulder pain is now limiting her activities. Aside from the
shoulder pain she is well. Physical examination reveals that Explanation: The presence of shoulder pain beginning during a
when the shoulder is flexed forwards passively this causes period a repetitive arm-raised activity (painting) with a positive
the patient pain. Neer’s test supports the diagnosis.

What is the most likely diagnosis?

A 73-year-old female presents to the GP with vaginal Fluconazole


discharge and pruritus that started 4 days ago. The discharge
is reported as white. The patient has type 2 diabetes mellitus Explanation: Candidiasis typically presents with a cottage-cheese
and hypertension for which she takes metformin and like white discharge and pruritus. It is more common in

17
enalapril. She has not been sexually active in the last 3 individuals who are immunocompromised, who have recently
months. She went through menopause at 51. She is completed a course of antibiotics or have increased oestrogen
haemodynamically stable and afebrile. Physical examination levels (e.g. COCP). Visualisation of hyphae and spores on wet
is performed and vaginal pH is assessed and found to be mount and a vaginal pH of <4.5 provide further support for the
<4.5. What is the most appropriate treatment? diagnosis. It may be treated with fluconazole.

20
Radial nerve
A 22-year-old female presents to ED following a motor
vehicle accident with left arm pain and decreased power. Explanation: The radial nerve runs through the spiral groove on
Physical examination reveals a left-sided wrist drop. An x- the humerus. Accordingly, mid-shaft humerus fractures may
ray subsequently shows a mid-shaft humerus fracture. result in damage to the radial nerve. The radial nerve innervates
the common extensors of the wrist in the posterior compartment
Which nerve is most likely to have been injured? of the forearm. Accordingly, this lesion may cause weakness of
D
wrist extension and wrist drop.
A 25-year-old male presents to the GP with back pain for the
NSAIDs
last 3-4 months. The pain is worst in the morning and better
TE
with activity. Over the last month he has also developed
Explanation: The presence of inflammatory back and hip pain
right hip pain. He has trialled going to a physiotherapist but
(better with activity) with sacroiliac tenderness and no other
has made no improvement. He has had no trauma to the back
features (such as bloody diarrhoea, venereal disease or psoriatic
or hip. He has also been feeling tired and generally unwell.
nail changes) to suggest another spondyloarthropathy
The patient has no significant past medical history. Vital
(enteropathic, reactive or psoriatic arthritis) support ankylosing
signs are stable and 37.4˚C. Physical examination reveals
spondylitis as the most likely diagnosis. Physiotherapy and
tenderness on palpation of the sacroiliac joints and a swollen
M

NSAIDs are the best initial treatment. If this does not provide
and tender right hip.
adequate relief tumour-necrosis factor inhibitors such as
infliximab or adalimumab may be used.
What is the best initial treatment for this patient?
AI

A 25-year-old male presents to the GP with a red and painful


left eye for the last 2 days. He has had no change to his
vision. The patient has no significant past medical history.
Vital signs are stable and 36.7˚C. Physical examination Behcet’s syndrome
reveals the left eye has a small pupil that reacts sluggishly to
light. The patient also has an ulcer on the scrotum and 2 Explanation: The combination of anterior uveitis, painful mouth
©

mouth ulcers. He reports that both the mouth and genital ulcer and genital ulcers make Behcet’s syndrome the most likely
ulcers are painful. The patient reports that he has not been diagnosis in this patient.
sexually active in the last 6 months.

What is the most likely diagnosis?

A 67-year-old male presents to the ED due to right knee pain NSAIDs


of 4 hours in duration. Physical examination reveals a
swollen erythematous right knee. Joint aspiration is Explanation: The joint aspiration results are diagnostic for
performed and reveals positively birefringent crystals with a pseudogout. First line treatment for pseudogout is an NSAID.
rhomboid shape. Oral prednisolone may be used if an NSAID is contraindicated.
Colchicine may be added if symptomatic relief has still not been
What is the most appropriate treatment? achieved.

18 © AIMTED 2017. All Rights Reserved. See website for disclaimer.


http://www.aimted.com

A 26-year-old female presents to the GP because of Propranolol


recurrent migraines. When she has a migraine she uses
eletriptan for alleviation of the acute headache. She is having Explanation: Propranolol and amitriptyline may be used as first
3-4 migraines per month despite avoiding all possible line options to prevent migraines when non-pharmacological
triggers. She is looking for a way to help prevent migraines. prevention strategies have failed and the patient is having regular
What is the most appropriate treatment? migraines.

A 23-year-old male presents to the GP with painful red eyes


that started 1 week ago. He has never had anything like this
before. Over the past 2 weeks he has also had pain in his
right shoulder, both knees and right ankle. The pain is worst
in the morning and improves with physical activity. The
Keratoderma blennorrhagicum
patient has a past medical history of gonorrhoea 1 month ago
that was treated with antibiotics. He reports that the
Explanation: This patient has presented with the classical triad of

17
antibiotics completely resolved his symptoms although it has
reactive arthritis: inflammatory arthritis, conjunctivitis and
started to be painful to urinate again over the past week.
urethritis with a preceding bout of venereal disease. Reactive
Vital signs are stable and 37.0˚C. Physical examination
arthritis is associated with keratoderma blennorrhagicum which
reveals the affected joints are swollen, erythematous and
typically effects the soles of the feet or palms of the hands.
tender to palpation with a limited active range of motion. A
rash of vesciulopustular lesions is found on the soles of his

20
feet.

What is the correct term for the rash on his feet?

A 30-year-old male is review by a cardiologist as he has


recently been diagnosed with Marfan’s syndrome. He is
concerned that he may have passed the disease onto his son. 50%
D
His wife has been tested and she does not have the
condition. Explanation: Marfan’s syndrome is inherited through an
autosomal dominant pattern.
TE
What is the likelihood that his child has inherited the
condition?
A 45-year-old male presents to the ED with following an
episode of vision loss in his right eye 2 hours ago. He
reports that suddenly and painlessly the vision in his right
Polyarteritis nodosa
eye decreased, before returning what he thinks was 30
M

seconds later. He reports that over the last week he has felt
Explanation: The combination of amaurosis fugax, systemic
feverish, tired and had muscle aches, as well as feeling
symptoms and a palpable purpuric rash makes it appear likely that
nauseas and vomiting several times. The patient has a past
this patient has a vasculitis. The amaurosis fugax and livedo
medical history of hypertension and hepatitis B, which is
reticularis support polyarteritis nodosa as a cause of this
managed with enalapril. Vital signs are stable (BP 150/92)
AI

vasculitis. His poorly controlled hypertension may indicate renal


and 37.8˚C. Physical examination reveals his skin has a
involvement. Polyarteritis nodosa may be associated with
mottled appearance, a palpable purpuric rash over his lower
hepatitis B.
limbs and 6/6 vision bilaterally.

What is the most likely diagnosis?


©

A 49-year-old female presents to the GP with intermittent


chest pain over the past 3 months. The chest pain occurs
Anti-centromere antibodies
after meals, particularly large meals, and is burning and
centrally located. She also reports that her fingers on both
Explanation: This patient has all of the features of CREST
hands have been hurting intermittently. The patient has no
syndrome (calcinosis, Raynaud’s phenomenon, oEsophageal
significant past medical history. Vital signs are stable and
dysmotility, sclerodactyly and telangiectasias) making limited
37.0˚C. Physical examination reveals telangiectasia over the
scleroderma the most likely diagnosis. Anti-centromere
hands as well as an unusual tight appearance of the skin over
antibodies are specific for CREST syndrome. Anti-Scl70
the fingers with white nodules over some extensor tendons.
antibodies are specific for diffuse scleroderma.
What is the most appropriate diagnostic investigation?

19 © AIMTED 2017. All Rights Reserved. See website for disclaimer.


http://www.aimted.com

A 6-week-old male is brought to the GP for a routine check- Descending aorta to pulmonary artery
up. On examination a continuous machinery murmur,
loudest at the left infraclavicular region, is heard. An Explanation: The ductus arteriosus connects these two structures
echocardiogram reveals a patent ductus arteriosus. in utero to enable blood flow to bypass the lungs. When
Eisenmenger’s syndrome occurs the left-to-right shunt of a patent
Which of the following best describes the blood flow ductus arteriosus may be reversed to a right-to-left shunt (in
through a patent ductus arteriosus? (in the absence of which case blood would then flow pulmonary artery to
Eisenmenger’s syndrome) descending aorta).
A 7-year-old male is brought to the GP by his parents with a
rash that started 5 days ago. The rash is not itchy or painful.
Reassurance
It is distributed under his right armpit and inner arm. He
reports that several other children on his football team have
Explanation: The presence of a largely asymptomatic rash of skin
recently developed similar rashes. The patient has no
coloured papules with central umbilication with a history of

17
significant past medical history and his vaccinations are up
recent contacts with a similar rash makes molluscum contagiosum
to date. He is haemodynamically stable and afebrile.
the most likely diagnosis. Molluscum contagiosum is self-limiting
Physical examination reveals a rash consisting of skin
and will usually resolve over several months with observation.
coloured papules with central umbilication. Otherwise
Lesions can also be curetted or frozen with liquid nitrogen to
physical examination is non-contributory.
remove them.

20
What is the most appropriate treatment for this patient?
A 4-year-old Indigenous female is brought to a rural GP by
her grandmother because she has had an itchy rash over her
hands and wrists that started 2 weeks ago. The patient
reports the itchiness is worst and night time and is stopping
her from sleeping. Several other family members have Sarcoptes scabei
developed a similar rash. The patient has no significant past
D
medical history and is up to date with her immunisations. Explanation: The presence of linear burrows is pathognomonic
She is haemodynamically stable and afebrile. Physical for scabies. The history of severe pruritus that is worst at night
examination reveals an erythematous papular rash over the time and is being experienced by other members of the same
TE

patient’s hands and wrists. Linear burrows can be seen in the household supports the diagnosis. Scabies is caused by infection
web spaces between the patient’s fingers. Otherwise with the parasitic mite sarcoptes scabei.
physical examination is non-contributory.

Infection with which of the following is most likely


responsible for this rash?
M

Congenital adrenal hyperplasia


A newborn baby is noted to have ambiguous genitalia. The
pregnancy was uncomplicated. Subsequent investigation
Explanation: Elevated 17-hydroxyprogesterone levels is
reveals elevated serum 17-hydroxyprogesterone levels.
diagnostic for 21-hydroxylase deficiency (which is the most
AI

common cause of congenital adrenal hyperplasia). This child


What is the most likely diagnosis?
would have a XX genotype.
A 12-year-old indigenous female presents to a rural GP with
her grandmother with joint pain that started 2 days ago. The
joint pain started in her left ankle several days ago but that
has now partially resolved and now her left knee is very
©

painful. This pain is making it difficult for her to walk. She


does not recall any trauma to either joint. She also reports Rheumatic fever
that she had a bad sore throat 4 weeks ago for which she did
not seek medical attention. The patient has no significant Explanation: This patient has several features of rheumatic fever
past medical history. The patient has never smoked and including migratory polyarthritis, erythema marginatum and a
doesn’t drink alcohol. She has a family history of ischaemic preceding throat infection 4 weeks ago. Being Indigenous the
heart disease and diabetes mellitus type 2. She is patient is in a high risk population for rheumatic fever.
haemodynamically stable and temperature is 38.2˚C.
Physical examination reveals subcutaneous nodules over her
elbows and a rash over the trunk that consists of a series of
non-pruritic pink rings. Otherwise physical examination is
non-contributory. What is the most likely diagnosis?

20 © AIMTED 2017. All Rights Reserved. See website for disclaimer.


http://www.aimted.com

Rectal biopsy
A male newborn is delivered and fails to pass meconium in
the first 48 hours of life and develops bilious vomiting. The
Explanation: Rectal biopsy is the gold standard for diagnosing
doctor considers Hirschsprung’s disease the most likely
Hirschsprung’s disease. Anorectal manometry, abdominal x-ray
diagnosis.
and barium enema may all have roles in the diagnosis of
Hirschsprung’s. Hirschsprung’s may occur with chromosomal
What investigation can confirm the presence of
abnormalities, such a Down’s syndrome, in which case
Hirschsprung’s disease most definitively?
karyotyping may be useful.
A 2-year-old female is brought to the ED with abdominal
pain that started 5 hours ago. The pain is severe and comes
in waves. During the episodes of pain the child flexes their Laparotomy
legs up to their abdomen. Between bouts of pain the child
appears almost entirely well. The pain is poorly localised. Explanation: The presence of colicky abdominal pain, vomiting
The child has vomited three times (partially digested food and stool with blood and mucus in this 1-year-old make

17
only) and passed stool containing blood and mucus. The intussusception the most likely diagnosis. The child is
patient has no significant past medical history and is up to haemodynamically unstable and there is evidence of peritonism
date for their immunisations. She is hypotensive, tachycardic so the most appropriate treatment laparotomy for surgical
and afebrile. Physical examination reveals a tender abdomen reduction/resection. A laparoscopic approach is not appropriate
that is rigid to palpation. Otherwise physical examination is for the emergency situation. In a haemodynamically stable child
non-contributory. an air or saline enema would be an appropriate initial approach.

20
What is the most appropriate definitive treatment?
A 20-year-old female presents to the GP with intermittent
wheezing and breathless over the past month. These >12% and 200mL reversibility in FEV1
episodes occur when exposed to cigarette smoke and on
exertion. At the time of presentation she is asymptomatic. Explanation: The diagnosis of asthma is supported by episodic
D
The patient has a past medical history of hayfever. The wheeze and breathlessness in response to exposure to triggers
patient has never smoked and consumes 1 standard drink of such as cigarette smoke and exertion. Asthma is diagnosed by
alcohol per day. Vital signs are HR 80, BP 124/80, RR 16, demonstrating an obstructive picture on spirometry that shows
O2 sats 99% RA and 37.1˚C. Physical examination reveals significant reversibility (>12% and 200mL reversibility in FEV1)
TE

no abnormalities. Spirometry is organised. FEV1/FVC ratio following administration of a short-acting bronchodilator. For
is < 0.7. Which of the following results would confirm the further information refer to the Australian Asthma Handbook.
most likely diagnosis?
Maternal chromosome 15 imprinting and paternal chromosome
A 22-month-old male is brought to the GP because his
15 deletion
parents are concerned about his development. He is found to
have both language and social delay. Physical examination
M

Explanation: The presence of delayed milestones likely indicating


reveals hypotonia and small hands and feet. His height is
intellectual disability, hypotonia, small hands and feet and
<3rd centile for age and his weight is between the 75th and
hyperphagia make Prader-Willi syndrome the most likely
90th centiles. His parents report that he eats voraciously.
diagnosis. The syndrome is due to a deletion on paternal
AI

chromosome 15 with an imprinted, and hence silenced,


This syndrome is caused by which genetic defect?
corresponding section on maternal chromosome 15.
A 5-year-old male is brought to the GP because his parents
are concerned that he is not growing. The patient’s height is Noonan syndrome
below the third centile for his age, as is his weight. His
previous measurements have always placed him an Explanation: The combination of short stature, a webbed neck and
©

equivalent distance below the third centile. He is currently pectus excavatum in a male is consistent with Noonan syndrome.
asymptomatic. The patient was born at term following an Noonan syndrome may occur in males and females and is quite
uncomplicated pregnancy and has no significant past similar to Turner syndrome (which only occurs in females).
medical history. Physical examination reveals a webbed Another difference is that Noonan syndrome is associated with
neck and pectus excavatum. pulmonary stenosis while Turner syndrome is associated with
aortic stenosis.
What is the most likely diagnosis?

21 © AIMTED 2017. All Rights Reserved. See website for disclaimer.


http://www.aimted.com

A 7-month-old male is brought to the GP because he has


been increasingly lethargic over the past 2-3 days. The
Haemoglobin electrophoresis
patient has no significant past medical history and the
pregnancy and delivery were uncomplicated. He is of
Explanation: The presence of pallor and hepatosplenomegaly in
Mediterranean descent and the family recalls members of
this Mediterranean patient with a family history of a ‘blood
their family having a ‘blood disorder’. Vital signs are stable
disorder’ suggests the possible diagnosis of beta thalassaemia.
and 37.0˚C. Physical examination reveals he is pale with
Haemoglobin electrophoresis is the diagnostic investigation of
hepatosplenomegaly.
choice for beta-thalassaemia.
What is the most appropriate diagnostic investigation?
A 9-year-old male presents to the GP with fatigue over the
past week. The patient has also had a headache, cough, sore
Autosomal dominant
throat and runny nose during this time. The patient has no
significant past medical history. The patient has no known

17
Explanation: The red blood cell appearance described on the
family history, although little is known about the medical
blood smear is that of a spherocyte. Spherocytes may occur in
history of the patient’s father. Vital signs are stable and
hereditary spherocytosis and autoimmune haemolytic anaemia
37.0˚C. Physical examination reveals pallor of the palmar
(warm or cold) (may also occur in neonates with ABO or rhesus
creases. CBE reveals low haemoglobin, normal MCV,
incompatibility). The negative direct Coombs test excludes
increased MCHC and elevated red cell distribution width.
autoimmune haemolytic anaemia. In hereditary spherocytosis

20
Blood smear shows small circular red blood cells lacking
haemolysis may be increased during viral illness. Hereditary
central pallor. Platelets and WCC are normal. Unconjugated
spherocytosis is inherited most commonly through an autosomal
bilirubin is slightly elevated. Direct Coombs test is negative.
dominant pattern in 3/4 of cases (autosomal recessive in 1/4 of
cases).
What is the most likely mode of inheritance of this
condition?
A 5-year-old male is brought to the GP by his parents
D
because his parents are concerned that he is having seizures.
His parents report that up to 20 times per day he has 10-20 Ethosuximide
second period during which he stops moving and is
unresponsive. He is unable to recall what is said to him Explanation: The described events involving a brief impairment
TE

during these staring spells. Vital signs are stable and 37.2˚C. in the level of consciousness (and no other motor components) is
Physical examination reveals normal developmental classical for childhood absence epilepsy. Since seizures may
milestones and no neurological deficit. An EEG during one occur up to 100 times per day this may affect school performance.
of these events reveals a generalised 3 Hz spike and wave An ictal EEG typically shows a generalised 3 Hz spike and wave
pattern. pattern. First line treatment is with ethosuximide.
M

What is the most appropriate treatment?


A 3-year-old male is brought to a GP because of
abnormalities in his gait. He has a past history of recurrent
respiratory tract infections. On examination he has an Ataxia-telangiectasia
AI

unsteady gait and appears at risk of falling. He also


demonstrates poor fine motor control when playing with Explanation: The combination of ataxia, possible
toys. Physical examination reveals dilated conjunctival immunodeficiency, telangectasias and progeric changes make
vessels and greying of his hair. ataxia-telangiectasia the most likely diagnosis in this patient.

What is the most likely diagnosis?


©

A 1-day-old female has a routine neonatal examination in Pavlik harness


hospital. The patient was the product of a pregnancy
complicated by multiple gestation (dichorionic, diamniotic Explanation: The presence of a positive Barlow’s test and
twins). The patient is haemodynamically stable and afebrile. Ortolani’s test confirm the diagnosis of developmental dysplasia
On physical examination there is a clunk when posterior of the hip (DDH). The limited hip abduction and asymmetrical
pressure is applied to the left hip when it is adducted. skin creases provide further support for the diagnosis. Under the
Abduction of the hip following this elicits a clicking sound. age of 6 months a Pavlik harness is the most appropriate
There is also limited abduction of the left hip and treatment for DDH. Over 6 months closed or open reduction will
asymmetrical gluteal creases. Otherwise physical be required, followed by the administration of a spica cast for 3-4
examination is non-contributory. months. Closed reduction is preferable as it is a less invasive
process, however, over the age of 18 months open reduction is
What is the most appropriate treatment? commonly required.

22 © AIMTED 2017. All Rights Reserved. See website for disclaimer.


http://www.aimted.com

A 15-year-old male is brought to the ED with left knee pain


that started 2 days ago. The patient reports that the pain
Pinning of the physis
started when he stumbled, but did not fall and was quite
sudden in onset. It has worsened in the past 2 days. The
Explanation: The x-ray findings confirm the likely diagnosis of
patient has a past medical history of obesity. Physical
slipped capital femoral epiphysis. This should be treated
examination reveals an antalgic gait. There are no
surgically by pinning the physis to prevent further slipping. The
abnormalities on examination of the left knee. The left hip
patient should be non-weight bearing on the affected leg until the
has limited internal rotation and abduction and is tender to
time of surgery. The Pavlik harness and open reduction under
palpation. Plain hip x-rays reveal disruption of Kline’s line
anaesthesia without fixation may be used for developmental
on the left femur.
dysplasia of the hip. Physiotherapy alone may be appropriate for
Perthes disease.
What is the most appropriate definitive management for this
patient?
A 5-year-old female is brought to the ED by her parents with

17
a fever that started 6 days ago ago. Her parents have tried Intravenous immunoglobulin and high dose aspirin
giving her Paracetamol, but this has not improved her fever.
She has also now developed a rash that is maculopapular in Explanation: The presence of a fever for 5 days, rash, changes in
appearance and covers most of the child’s trunk. The patient the extremities, conjunctivitis, lymphadenopathy and oral mucosa
has no significant past medical history and is up to date on changes meet the diagnostic criteria for Kawasaki disease.
her immunisations. She is haemodynamically stable and has Standard treatment of Kawasaki disease during this acute phase

20
a temperature of 39.3˚C. Physical examination reveals right- involves intravenous immunoglobulin (IVIG) and high dose
sided cervical lymphadenopathy, oedema of her hands and aspirin to prevent the formation of coronary artery aneurysms.
bilateral conjunctivitis. Otherwise physical examination is Corticosteroids may be added to this regimen in some centres.
non-contributory. Corticosteroids and cyclophosphamide may be used in cases that
are refractory to IVIG, however, IVIG is the first line treatment.
What is the best definitive treatment for this patient?
A 3-year-old male is brought to the GP by his parents with
D
Hypoplastic left ventricle
unusual episodes in which the child squats, goes silent and
then cries. These episodes started 1 month ago. The patient
Explanation: The presence of cyanotic episodes associated with
has no known past medical history. However, the mother
crouching is typical of tet spells. Tet spells in conjunction with a
TE
had limited antenatal care during her pregnancy. The child’s
systolic ejection murmur heard loudest at the upper left sternal
vaccinations are up to date. Physical examination reveals
border and a loud S2 in a child of this age makes tetralogy of
clubbing, a single loud S2 and a systolic ejection murmur
Fallot the most likely diagnosis. Tetralogy of Fallot is congenital
heard loudest at the left upper sternal border. Otherwise
heart deformity in which there is right ventricular outflow tract
physical examination is non-contributory.
obstruction (pulmonary stenosis), right ventricular hypertrophy, a
ventricular septal defect and an overriding aorta. A hypoplastic
Which of the following is NOT a feature of the most likely
M

left ventricle is a feature of hypoplastic left heart syndrome.


diagnosis?
Decrease in prostaglandin E2

A premature infant is born with a patent ductus arteriosus. Explanation: The role that prostaglandin E2 plays in maintaining
AI

The intern is asked to explain to the parents why the patency of the ductus arteriosus is the reason that, should the
indomethacin will be used to close the duct. The infant’s duct remain patent and need to be closed, a non-selective
parents ask why the duct has not closed. cyclooxygenase (COX) inhibitor, such as indomethacin or
ibuprofen, may be used to close the duct. COX inhibitors decrease
What is the normal stimulus for the closure of the ductus prostaglandin production. Conversely prostaglandin E2 can be
arteriosus? given to maintain the patency of the duct if an infant has a
©

congenital heart malformation that is duct dependent (such as


transposition of the great arteries).

23 © AIMTED 2017. All Rights Reserved. See website for disclaimer.


http://www.aimted.com

Galactose-1-phosphate uridyl transferase (GALT)


A 10-day-old female is brought to the ED by her parents for
Explanation: The presence of vomiting, lethargy, hypotonia,
her recurrent vomiting and lethargy. The baby was delivered
jaundice, hepatomegaly and cataracts within days of birth support
at term after an uncomplicated pregnancy and is being
the diagnosis of galactosaemia. Deficiency of galactose-1-
exclusively breastfed. The patient is haemodynamically
phosphate uridyl transferase is present in galactosaemia.
stable and afebrile. Physical examination reveals hypotonia,
jaundice, hepatomegaly and bilateral cataracts. Otherwise
Deficiency of branched-chain alpha-keto acid dehydrogenase
physical examination is non-contributory.
complex is present in maple syrup urine disease. Deficiency of
hypoxanthine-guanine phosphoribosyltransferase may result in
Deficiency of which enzyme is most likely present in this
Lesch-Nyhan syndrome. Myophosphorylase deficiency causes
patient?
McArdle’s syndrome. Phenylalanine hydroxylase deficiency
causes phenylketonuria.
Angelman syndrome

17
An 18-month-old male is brought to the GP with language
and social developmental delay. He appears to be a happy
Explanation: Angelman syndrome typically involves intellectual
child, who breaks into laughter at random intervals. He has
disability, happy disposition, random laughing episodes and pale
white hair and pale blue eyes.
complexion with blue eyes. The syndrome is due to a deletion on
maternal chromosome 15 with an imprinted, and hence silenced,
What is the most likely diagnosis?

20
corresponding section on paternal chromosome 15.
A 6-month-old female is brought to the ED because she has
Streptococcus pneumoniae
been crying and been unable to be settled for 8 hours. The
patient has no significant past medical history and the
Explanation: The presence of irritability, jaundice, splenomegaly
pregnancy and delivery were uncomplicated. Vital signs are
and dactylitis in this patient supports the diagnosis of sickle cell
stable and 37.1˚C. Physical examination reveals jaundice of
anaemia. Sickle cell anaemia leads to autosplenectomy which
D
the sclera, swelling of the hands and feet and splenomegaly.
predisposes to infection with encapsulated organisms, such as
Streptococcus pneumoniae, Haemophilus influenzae type B and
Patients with this condition are predisposed to infection with
salmonella.
which bacteria?
TE

A 5-year-old male is brought to a rural GP with a sore throat


that started 4 days ago. The child has also complained of
Gram positive cocci that forms chains
having muscle aches and has vomited twice. The patient has
also developed a rash over the past 36-48 hours. The patient
Explanation: The red papules on a white tongue is often referred
has no significant past medical history. Vital signs are stable
to as a strawberry tongue is classical for scarlet fever or Kawasaki
and 38.4˚C. Physical examination also reveals enlarged
disease. The enlarged tonsils with exudate support the diagnosis
M

tonsils with a white exudate. His tongue has numerous


of scarlet fever. The lack of involvement of the conjunctivae and
erythematous papules on a white base. The rash involves his
lips helps to distinguish the presentation from Kawasaki disease.
neck, trunk and extremities. The rash is erythematous and
A rapid strep test, throat culture and/or anti-streptolysin O titre
maculopapular. He has tender cervical lymphadenopathy.
may be performed to confirm the diagnosis. The rash is caused by
AI

Streptococcus pyogenes (gram positive cocci that forms chains).


What is the gram stain of bacteria that is responsible for this
condition?
A 7-year-old male is brought to the GP by his parents
Perthes disease
because he has developed a limp that started 2 days ago. The
patient reports that he has had some mild aching pain in his
Explanation: The presence of a limp with only mild pain
©

right hip over the past week. He has had no trauma to the hip
accompanied by limited internal rotation and abduction in this 7-
and no pain anywhere else. He is otherwise systemically
year-old male sprinting athlete makes perthes disease the most
well. The patient has no significant past medical history. He
likely diagnosis. Developmental dysplasia of the hip typically
is haemodynamically stable and afebrile. Physical
presents years earlier than this, although it may be considered a
examination reveals moderately decreased range of motion
differential. Septic arthritis would typically present more acutely
on internal rotation and abduction of the right hip. The hip
than this with a refusal to weight bear, grossly inflamed joint,
itself is not visibly inflamed, warm or tender to palpation.
severely decreased range of motion and a fever. Slipped capital
He is noted to have an antalgic gait. Otherwise physical
femoral epiphysis typically presents in older children who are
examination is non-contributory. The patient is concerned
overweight or obese and has pain as a more prominent feature.
because he has an athletics event coming up in 1 month in
Transient synovitis of the hip is a diagnosis of exclusion; it would
which he will be competing in the 100m sprint.
be supported by a preceding history of an upper respiratory tract
infection.
What is the most likely diagnosis?

24 © AIMTED 2017. All Rights Reserved. See website for disclaimer.


http://www.aimted.com

Placental abruption
A 33-year-old G3P2 female at 34 weeks pregnant presents to
the ED with dark red vaginal bleeding and abdominal pain Explanation: The presence of constant abdominal pain and
that started 30 minutes ago. The pain is constant and vaginal bleeding following trauma in this woman who is 34
worsening. The pregnancy has been complicated by pre- weeks pregnant makes placental abruption the most likely
existing diabetes mellitus type 2 and cocaine use. She is diagnosis. The ultrasound is performed largely to exclude
haemodynamically stable and afebrile. Physical examination placenta praevia and is NOT sensitive for placental abruption.
reveals abdominal tenderness and a firm uterus. CTG Therefore, the negative finding does not exclude placental
demonstrates no evidence of fetal distress. A transabdominal abruption. Placenta praevia, vasa praevia and ectropion are
ultrasound reveals no abnormalities. What is the most likely typically painless. Cervical cancer would be unlikely to cause
diagnosis? abdominal pain and such an acute presentation of vaginal
bleeding.

17
20
D
TE
M
AI
©

25 © AIMTED 2017. All Rights Reserved. See website for disclaimer.


http://www.aimted.com

You might also like